How do I determine where a function has a removable and a jump discontinuity?

Click For Summary
To determine removable and jump discontinuities for the piecewise function, one must analyze the limits as x approaches 1 from both sides. A removable discontinuity occurs when the left-hand limit equals the right-hand limit, but the function value at x=1 does not match this limit. Conversely, a jump discontinuity exists when both one-sided limits exist but are not equal. The discussion emphasizes the need to find the appropriate value of 'b' to achieve these conditions. Understanding these concepts is crucial for solving the problem effectively.
rowkem
Messages
48
Reaction score
0

Homework Statement



I am given the following function, piecewise:

f(x) = (-x+b) (x<1)
3 if x=1
(-12/(x-b))-1 (x>1, x=/b)

I am asked:

1) For what value(s) of 'b' does 'f' have a removable discontinuity at 1?
2) For what value(s) of 'b' does 'f' have a (finite) jump discontinuity at 1? Write your answer in interval notation.

Homework Equations



X

The Attempt at a Solution



Honestly, I have to clue where to start; especially in regards to the removable discontinuity. I tried making the functions equal each other for the jump discontinuity but, that was way out as far as I can tell.

--------------------------

If I could at least be given a starting point to go from, it would be appreciated. I'm not asking for the answers but, I'm so lost and just staring at this thing isn't helping. Thanks,

RK
 
Physics news on Phys.org
If a function has a removable discontinuity at x = a, that means these things.

<br /> \begin{align*}<br /> \lim_{x \to a^+} f(x) &amp; = \lim_{x \to a^-} f(x)\\<br /> \text{ so } &amp; \lim_{x \to a} f(x) \text{ exists}\\<br /> f(a) &amp; \text{ is defined} \\<br /> f(a) &amp; \ne \lim_{x \to a^+} f(x)<br /> \end{align*}<br />

All the limits above are finite. Basically, if there is a removable discontinuity at x = a, the function has almost everything needed to be continuous there, but the function value is "wrong". Here is an example.

<br /> f(x) = \begin{cases}<br /> 3x+5 &amp; \text{ if } x \ne 10\\<br /> 20 &amp; \text{ if } x = 10 <br /> \end{cases}<br />

Here the limit at 10 is equal to 35 (because both one-sided limits equal 35), but f(10) = 20, so the function is not continuous there. We say there is a removable discontinuity at 10 because we can do this: make f continuous merely by doing this:

<br /> F(x) = \begin{cases}<br /> 3x + 5 &amp; \text{ if } x \ne 10\\<br /> 35 &amp; \text{ if } x = 10 <br /> \end{cases}<br />

i.e. - we remove the discontinuity by redefining the function at the problem point, in order to make the function continuous there.

A function has a jump discontinuity at a spot if the two one-sided limits both exist but are not equal. Here is an example - the jump discontinuity is at x = 2.

<br /> w(x) = \begin{cases}<br /> 2x - 1 &amp; \text{ if } x &lt;=2\\<br /> 10x + 1 &amp; \text{ if } x &gt; 2<br /> \end{cases}<br />

The limit from the left is 3, the limit from the right is 21. If you were to view the graph of w you would see a 'jump' in the two portions of the graph at x = 2. What you need to do is find the values of a and b to make your function have the types of behavior discussed here. Good luck.
 
If you could please relate that back to my question, it would be appreciated. Again, no answers but, I'm still a little confused and don't know where to start.
 
statdad said:
If a function has a removable discontinuity at x = a, that means these things.

<br /> \begin{align*}<br /> \lim_{x \to a^+} f(x) &amp; = \lim_{x \to a^-} f(x)\\<br /> \text{ so } &amp; \lim_{x \to a} f(x) \text{ exists}\\<br /> f(a) &amp; \text{ is defined} \\<br /> f(a) &amp; \ne \lim_{x \to a^+} f(x)<br /> \end{align*}<br />


luck.
It is also the case where f(a) is not defined but the overall limit as x-->a exists. The way one can remove this is by defining the function at x=a.

P.S. I am sure you know this, but this is adressed to the OP.
 
Question: A clock's minute hand has length 4 and its hour hand has length 3. What is the distance between the tips at the moment when it is increasing most rapidly?(Putnam Exam Question) Answer: Making assumption that both the hands moves at constant angular velocities, the answer is ## \sqrt{7} .## But don't you think this assumption is somewhat doubtful and wrong?

Similar threads

Replies
2
Views
2K
Replies
2
Views
2K
  • · Replies 4 ·
Replies
4
Views
2K
  • · Replies 3 ·
Replies
3
Views
2K
Replies
4
Views
4K
  • · Replies 13 ·
Replies
13
Views
3K
  • · Replies 3 ·
Replies
3
Views
4K
Replies
2
Views
2K
  • · Replies 9 ·
Replies
9
Views
4K
  • · Replies 10 ·
Replies
10
Views
2K